Đến nội dung

Hình ảnh

Topic về số học, các bài toán về số học.

- - - - - topic số học hay tuyệt

  • Please log in to reply
Chủ đề này có 171 trả lời

#101
Juliel

Juliel

    Thượng úy

  • Thành viên
  • 1240 Bài viết

Bài 46 : Cho các số nguyên $x,y,z$ thỏa mãn $\frac{x}{y}-\frac{y}{z}+\frac{z}{x}=9$. Chứng minh rằng $\sqrt[3]{xyz}\in \mathbb{Z}$


Đừng rời xa tôi vì tôi lỡ yêu người mất rồi !
 

Welcome to My Facebook !


#102
nhatduy01

nhatduy01

    Trung sĩ

  • Thành viên
  • 132 Bài viết

Bài 46 : Cho các số nguyên $x,y,z$ thỏa mãn $\frac{x}{y}-\frac{y}{z}+\frac{z}{x}=9$. Chứng minh rằng $\sqrt[3]{xyz}\in \mathbb{Z}$

Gọi $(x,y,z)=d\Rightarrow x=dx_{0},y=dy_{0},z=dz_{0}$ với $x_{0},y_{0},z_{0}\in \mathbb{Z}$,và $(x_{0},y_{0},z_{0})=1$.

Khi đó 

              $\frac{x_{0}}{y_{0}}-\frac{y_{0}}{z_{0}}+\frac{z_{0}}{x_{0}}=9$   (1)

             $\sqrt[3]{xyz}=d\sqrt[3]{x_{0}y_{0}z_{0}}\in \mathbb{Z}\Leftrightarrow \sqrt[3]{x_{0}y_{0}z_{0}}\in \mathbb{Z}$

Nhận xét bộ $(x_{0},y_{0},z_{0})$: $x_{0}y_{0}z_{0}=\pm1$ không thể thỏa mãn (1) nên $x_{0}y_{0}z_{0}\neq \pm 1$.

Gọi p là một ước nguyên tố bất kì của $x_{0}y_{0}z_{0}$.Vì

               (1)$\Leftrightarrow x_{0}^{2}z_{0}-y_{0}^{2}x_{0}+z_{0}^{2}y_{0}=9x_{0}y_{0}z_{0}$   (2)

 và $(x_{0},y_{0},z_{0})=1$ nên p chỉ là ước của đúng 2 trong 3 số $x_{0},y_{0},z_{0}$.

Không giảm tổng quát ,giả sử p là ước của $x_{0},y_{0}$ và không là ước của $z_{0}$.

     Gọi m,n lần lượt là số mũ của p trong phân tích tiêu chuẩn của $x_{0},y_{0}$$(m,n\in \mathbb{Z}^{+})$.

Ta xét hai trường hợp

          i) Nếu $n\geq 2m+1$:ta có $p^{2m+1}$ là ước của $z_{0}^{2}y_{0},y_{0}^{2}x_{0},9x_{0}y_{0}z_{0}$ nên từ (2) suy ra $p^{2m+1}$ là ước của $x_{0}^{2}z_{0}$,mà $x_{0}^{2}$ không chia hết cho $p^{2m+1}$ nên p là ước của $z_{0}$   (mâu thuẫn).

          ii) Nếu $n\leq 2m-1$ $\Leftrightarrow n+1\leq 2m$:ta có $p^{n+1}$ là ước của  $x_{0}^{2}z_{0},y_{0}^{2}x_{0},9x_{0}y_{0}z_{0}$

nên từ (2) suy ra $p^{m+1}$ là ước của $z_{0}^{2}y_{0}$,mà $y_{0}$ không chia hết cho $p^{n+1}$ nên p là ước của $z_{0}$(mâu thuẫn)

Vậy $n=2m$ nên $3m$ là số mũ của p trong phân tích tiêu chuẩn của $x_{0}y_{0}z_{0}$.

Suy ra $x_{0}y_{0}z_{0}=\prod p_{i}^{3m_{i}}$(với $p_{i},i=1...k$,là các ước nguyên tố của $x_{0}y_{0}z_{0}$)

  Vậy $\sqrt[3]{x_{0}y_{0}z_{0}}\in \mathbb{Z}$. 



#103
Kool LL

Kool LL

    Sĩ quan

  • Thành viên
  • 370 Bài viết

Bài 27: Cho 2 bộ ba số nguyên dương a, b, c và d, e, f sao cho $(a,b,c)=1;(d,e,f)=1$ (1) và thoả mãn đẳng thức $\frac{1}{a}+\frac{1}{b}=\frac{1}{c}$; $\frac{1}{d}+\frac{1}{e}=\frac{1}{f}$ (2). Chứng minh rằng 2(a + b + d + e) là tổng bình phương của hai số tự nhiên.  

 

Nếu $a=b$ thì $\overset{(2)}{\Rightarrow} a=b=2c$ $\overset{(1)}{\Rightarrow} 1=(a,b,c)=(2c,2c,c)=c$ $\Rightarrow a=b=2\Rightarrow a+b=4=2^2=(c+1)^2$.

Giả sử $a>b$ thì $a>2c>b>c\Rightarrow a-c>c>b-c\ge1$.

Gọi $d=(b,c)$ thì $b=db_1; c=dc_1; (b_1,c_1)=1$.

$(2)\Rightarrow ab=c(a+b)\Rightarrow ab_1=c_1(a+db_1)\Rightarrow a\vdots b_1, a\vdots c_1\Rightarrow a\vdots b_1c_1$ $\Rightarrow a=k.b_1c_1\Rightarrow k(b_1-c_1)=d\Rightarrow k|d|b,c$ và $k|a\Rightarrow k=1$

$\Rightarrow a=b_1c_1; b_1-c_1=d\Rightarrow b-c=d^2\Rightarrow a+b=b_1c_1+c+d^2=b_1c_1+(b_1-c_1)c_1+$$(b_1-c_1)^2=b_1^2$

Vậy ta luôn có $a+b$ là số chính phương $m^2$. Tương tự với $d+e=n^2$.

Suy ra $2(a+b+d+e)=2(m^2+n^2)=(m+n)^2+(m-n)^2$ là tổng bình phương của hai số tự nhiên. (đpcm)


Bài viết đã được chỉnh sửa nội dung bởi Kool LL: 24-08-2013 - 05:57


#104
Kool LL

Kool LL

    Sĩ quan

  • Thành viên
  • 370 Bài viết

Giải bài 27 (đã hết hạn bỏ bài  :icon6: ): Trước hết ta sẽ chứng minh rằng a + b và c + d đều là số chính phương.

Vì (a, b, c) = 1 tức là a, b, c ko có nhân tử chung thế nên ta hoàn toàn có thể giả sử $b\vdots c,(a,c)=1$. ...

 

Căn cứ nào mà lại có thể giả sử như thế !!? Bạn chứng minh được điều đó không ??



#105
Kool LL

Kool LL

    Sĩ quan

  • Thành viên
  • 370 Bài viết

Bài này mình chỉ giải được trên tập nghiệm tự nhiên nên trình bày cách giải ra để mọi người cùng góp ý, nếu ai giải được trên tập nghiệm nguyên thì bổ sung cho bài của mình nhé !

 

$PT\Leftrightarrow z(xy+2)=x^{2}+2\Rightarrow z=\frac{x^{2}+2}{xy+2}\in N$

$\Rightarrow x+\frac{2(y-x)}{xy+2}\in Z\Rightarrow 2(y-x)=k(xy+2)$ với $k$ tự nhiên

 

  • Xét $x=1$ ta có $yz=3-2z\Leftrightarrow z(y+2)=3\Rightarrow y=z=1$
  • Xét $x\geq 2$ :

Nếu $k = 0$ thì $x^{2}+2=0$ (loại)

Nếu $k\geq 1$ thì $2(y-x)\geq xy+2\Leftrightarrow (x-2)(y+2)+6\leq 0$

Nhưng $x\geq 2,y\geq 0\Rightarrow (x-2)(y+2)+6>0$

Phương trình vô nghiệm

Vậy : Phương trình có nghiệm tự nhiên duy nhất $(1 ; 1; 1)$

 

$k\in\mathbb{Z}$ mà, sao lại cho $\in\mathbb{N}$ luôn vậy !!!

 

$k=0$ thì $x=y$ và PT có VSN dạng $(t,t,1)$ mà !!!



#106
Kool LL

Kool LL

    Sĩ quan

  • Thành viên
  • 370 Bài viết

Bài 29

Cho $p_{n}$ là số nguyên tố thứ $n$. Chứng minh rằng:

a.$p_{n}>2n$ với mọi $n>4$

b.$p_{n}>3n$ với mọi $n>11$

 

a) Ta CM bang pp qui nạp theo $n>4$.

  • $n=5$: $p_5=11>2.5=10$ (đúng).
  • G/s bài toán đúng đến $n=k, (k>4)$, tức là ta có $p_n>2n \forall n=\overline{5..k}$.
  • Xét n=k+1: Do $p_{k+1} > p_k$ là hai số nguyên tố lẻ liên tiếp nên $p_{k+1}\ge p_k+2$.

Dùng giả thiết qui nạp, ta có $p_k>2k$. Suy ra $p_{k+1}>2k+2=2(k+1)$

Như vậy bài toán cũng đúng với $n=k+1$.

Theo nguyên lý quy nạp suy ra bài toán đúng $\forall n>4$. (đpcm)

 

b)  Ta CM bang pp qui nap theo $n>11$.

  • $n=12$: $p_{12}=37>3.12=36$ (đúng).
  • G/s bài toán đúng đến $n=k, (k>11)$, tức là ta có $p_n>3n \forall n=\overline{12..k}$.
  • Xét n=k+1: Do $p_{k+1} > p_k$ là hai số nguyên tố lẻ lien tiếp nên $p_{k+1}\ge p_k+2$.

Dùng giả thiết qui nạp, ta có $p_k>3n$, tức là $p_k\ge 3n+1$. Suy ra $p_{k+1}\ge 3k+1+2=3(k+1)$.

Dấu $"="$ không thể xảy ra vì khi đó $p_{k+1}\vdots3$ (mâu thuẫn với $p_{k+1}$ là số nguyên tố).

Vậy $p_{k+1}>3(k+1)$. Bài toán cũng đúng với $n=k+1$.

Theo nguyên lý qui nạp suy ra bài toán đúng $\forall n>11$. (đpcm)


Bài viết đã được chỉnh sửa nội dung bởi Kool LL: 24-08-2013 - 16:17


#107
Kool LL

Kool LL

    Sĩ quan

  • Thành viên
  • 370 Bài viết

Bài 31:

 Cho số tự nhiên y .Chứng minh tồn tại vô số nguyên tố p sao cho d08057d8d86e79b7b0888b1fc06d3ef6_4.0pt.p và 72e8d7b77259e1ece54a452e313d04d0_4.0pt.p.

 

Viết đề kiểu này khó hiểu quá! Phát biểu lại ý của đề bài cho dễ hiểu hơn đây :

"Cho trước số tự nhiên $y$. CMR : Có vô số số tự nhiên $n$ để số $(2^ny+1)$ có ước nguyên tố dạng $4k+3$."

 


Bài viết đã được chỉnh sửa nội dung bởi Kool LL: 24-08-2013 - 16:55


#108
Juliel

Juliel

    Thượng úy

  • Thành viên
  • 1240 Bài viết

Bài 47 : Giải phương trình nghiệm nguyên dương $p^{x}-y^{p}=1$ trong đó $p\in \mathbb{P}$.


Đừng rời xa tôi vì tôi lỡ yêu người mất rồi !
 

Welcome to My Facebook !


#109
Zaraki

Zaraki

    PQT

  • Phó Quản lý Toán Cao cấp
  • 4273 Bài viết

Bài 47 : Giải phương trình nghiệm nguyên dương $p^{x}-y^{p}=1$ trong đó $p\in \mathbb{P}$.

Lời giải. Với $p=2$ thì phương trình tương đương với $2^x=y^2+1$. 

Nếu $x \ge 2$ thì $4|2^x$ nên $y^2 \equiv 3 \pmod{4}$, mâu thuẫn. Vậy $x=1$. Khi đó $y=1$.

Với $p \ge 3$ thì $p$ lẻ, phương trình tương đương $p^x=y^p+1$. Ta suy ra $p|y+1$. Áp dụng bổ đề LTE ta có $$v_p(y^p+1)=v_p(p)+v_p(y+1)=1+v_p(y+1)=x$$

Do đó $y+1=p^{x-1}$. Với $x=1$ thì $y=0$, mâu thuẫn. Vậy $x \ge 2$.

Ta suy ra $p^{x-1} \ge p \Leftrightarrow y+1 \ge \frac{y^p+1}{y+1} \Leftrightarrow (y+1)^2 \ge y^p+1 \qquad (1)$.

Tuy nhiên ta lại có $(y+1)^2 < y^p+1$ với $p,y \ge 3$. Vậy nên $(1)$ xảy ra khi $y=2$ hoặc $y=1$.

Với $y=1$ thì $p=2$, mâu thuẫn.

Với $y=2$ thì $p^{x-1}=3 \Rightarrow x=2,p=3$.

Vậy phương trình có nghiệm nguyên dương $\boxed{ (x,y,p)=(1,1,2),(2,2,3)}$.


Discovery is a child’s privilege. I mean the small child, the child who is not afraid to be wrong, to look silly, to not be serious, and to act differently from everyone else. He is also not afraid that the things he is interested in are in bad taste or turn out to be different from his expectations, from what they should be, or rather he is not afraid of what they actually are. He ignores the silent and flawless consensus that is part of the air we breathe – the consensus of all the people who are, or are reputed to be, reasonable.

 

Grothendieck, Récoltes et Semailles (“Crops and Seeds”). 


#110
Juliel

Juliel

    Thượng úy

  • Thành viên
  • 1240 Bài viết

Bài 48 : Cho $a,b$ là các số tự nhiên lẻ, $p$ là số nguyên tố lẻ sao cho $a+b$ chia hết cho $p$ và $a-b$ chia hết cho $p-1$. Chứng minh rằng $a^{b}+b^{a}$ chia hết cho $2p$ và $a^{a}+b^{b}$ chia hết cho $2p$.


Bài viết đã được chỉnh sửa nội dung bởi Juliel: 01-09-2013 - 22:00

Đừng rời xa tôi vì tôi lỡ yêu người mất rồi !
 

Welcome to My Facebook !


#111
Zaraki

Zaraki

    PQT

  • Phó Quản lý Toán Cao cấp
  • 4273 Bài viết

Bài 48 : Cho $a,b$ là các số tự nhiên lẻ, $p$ là số nguyên tố lẻ sao cho $a+b$ chia hết cho $p$ và $a-b$ chia hết cho $p-1$. Chứng minh rằng $a^{b}+b^{a}$ chia hết cho $2p$ và $a^{a}+b^{b}$ chia hết cho $2p$.

Bài này nhìn đẹp thiệt!!  :lol:

Lời giải. Không mất tính tổng quát, giả sử $a \ge b$. Nếu $p|a,p|b$ thì hiển nhiên ta có điều phải chứng minh. Nếu $p \nmid a,p \nmid b$ thì ta có $a^b+b^a= (a^b+b^b)+ b^{b} (b^{a-b}-1)$.

Vì $b$ lẻ nên $p|a+b|a^b+b^b$. Áp dụng Định lý Fermat nhỏ thì $b^{p-1}-1 \equiv 0 \pmod{p}$. Mà $p-1|a-b$ nên $b^{p-1}-1|p^{a-b}-1$. Do đó $p|p^{a-b}-1$. Vậy $a^b+b^a$ chia hết cho $2p$.

Hoàn toàn tương tự thì $a^a+b^b= (b^b+a^b)+ a^b(a^{a-b}-1)$ cũng chia hết cho $2p$.


Bài viết đã được chỉnh sửa nội dung bởi Jinbe: 02-09-2013 - 07:54

Discovery is a child’s privilege. I mean the small child, the child who is not afraid to be wrong, to look silly, to not be serious, and to act differently from everyone else. He is also not afraid that the things he is interested in are in bad taste or turn out to be different from his expectations, from what they should be, or rather he is not afraid of what they actually are. He ignores the silent and flawless consensus that is part of the air we breathe – the consensus of all the people who are, or are reputed to be, reasonable.

 

Grothendieck, Récoltes et Semailles (“Crops and Seeds”). 


#112
Zaraki

Zaraki

    PQT

  • Phó Quản lý Toán Cao cấp
  • 4273 Bài viết

Đóng góp bài:

Bài 49. Tìm số nguyên tố $p$ thoả mãn $p^3-4p+9$ là số chính phương.


Discovery is a child’s privilege. I mean the small child, the child who is not afraid to be wrong, to look silly, to not be serious, and to act differently from everyone else. He is also not afraid that the things he is interested in are in bad taste or turn out to be different from his expectations, from what they should be, or rather he is not afraid of what they actually are. He ignores the silent and flawless consensus that is part of the air we breathe – the consensus of all the people who are, or are reputed to be, reasonable.

 

Grothendieck, Récoltes et Semailles (“Crops and Seeds”). 


#113
Juliel

Juliel

    Thượng úy

  • Thành viên
  • 1240 Bài viết

Đóng góp bài:

Bài 49. Tìm số nguyên tố $p$ thoả mãn $p^3-4p+9$ là số chính phương.

Đặt $$p^{3}-4p+9=t^{2}\qquad(t\in \mathbb{N})$$

Biến đổi thành : $$p(p^{2}-4)=(t-3)(t+3)(1)\Rightarrow p|t-3\vee p|t+3$$

$\blacktriangledown$ Trường hợp 1 : Nếu $p|t-3$. Đặt $t-3=pk\qquad(k\in \mathbb{N})$

Khi đó thay vào $(1)$ : $$p(p^{2}-4)=pk.(t+3)\Leftrightarrow k(t+3)=p^{2}-4\Rightarrow p^{2}=kt+3k+4$$

Mặt khác có $$(t-3)^{2}=p^{2}k^{2}\Rightarrow t^{2}-6t+9=k^{2}(kt+3k+4)\Leftrightarrow t^{2}-t(6+k^{3})+9-3k^{3}-4k^{2}=0$$

Coi đây là phương trình bậc hai ẩn $t$, điều kiện cần để tồn tại nghiệm nguyên của phương trình là :

$$\Delta =(6+k^{3})^{2}-4(9-3k^{3}-4k^{2})=k^{6}+24k^{3}+16k^{2}=k^{2}(k^{4}+24k+16)$$ 

Là một số chính phương. Muốn vậy thì $k^{4}+24k+16$ phải là một số chính phương.

Mặt khác với $k>3$, ta dễ chứng minh được $(k^{2})^{2}<k^{4}+24k+16<(k^{2}+4)^{2}$. Suy ra các trường hợp 

+) $k^{4}+24k+16=(k^{2}+1)^{2}\Leftrightarrow 2k^{2}-24k-15=0$ (loại)

+) $k^{4}+24k+16=(k^{2}+2)^{2}\Leftrightarrow k^{2}-6k-3=0$ (loại)

+) $k^{4}+24k+16=(k^{2}+3)^{2}\Leftrightarrow 6k^{2}-24k-7=0$ (loại)

Do đó phải có $k\leq 3$, thử trực tiếp được $k=3$ thỏa mãn. Từ đó tìm được $t=36,p=11$.

$\blacktriangledown$ Trường hợp 2 : Nếu $p|t+3$. Đặt $t+3=pk\qquad(k\in \mathbb{N})$.

Khi đó thay vào $(1)$, được : $$p(p^{2}-4)=pk.(t-3)\Rightarrow p^{2}=kt-3k+4$$

Mặt khác, ta có $$(t+3)^{2}=p^{2}k^{2}\Leftrightarrow t^{2}+6t+9=k^{2}(kt-3k+4)\Leftrightarrow t^{2}+(6-k^{3})t+9+3k^{3}-4k^{2}=0$$

Coi đây là phương trình bậc hai ẩn $t$, điều kiện cần để tồn tại nghiệm nguyên của phương trình trên là :

$$\Delta =(6-k^{3})^{2}-4(9+3k^{3}-4k^{2})=k^{2}(k^{4}-24k+16)$$

Là số chính phương. Muốn vậy thì $k^{4}-24k+16$ phải là số chính phương.

Mặt khác ta dễ chứng minh được rằng với $k>3$ thì $(k^{2}-4)^{2}<k^{4}-24k+16<(k^{2})^{2}$. Suy ra các trường hợp :

+) $k^{4}-24k+16=(k^{2}-1)^{2}\Leftrightarrow 2k^{2}-24k+15=0$ (loại)

+) $k^{4}-24k+16=(k^{2}-2)^{2}\Leftrightarrow k^{2}-6k+3=0$ (loại)

+) $k^{4}-24k+16=(k^{2}-3)^{2}\Leftrightarrow 6k^{2}-24k+7=0$ (loại)

Do đó $k\leq 3$. Thử trực tiếp thấy $k=3$ thỏa mãn, khi đó $t=3;18$, tương ứng $p=2;7$

 

Kết luận$\boxed{p\in \left \{ 2;7;11 \right \}}$


Bài viết đã được chỉnh sửa nội dung bởi Juliel: 02-09-2013 - 20:31

Đừng rời xa tôi vì tôi lỡ yêu người mất rồi !
 

Welcome to My Facebook !


#114
bachhammer

bachhammer

    Thiếu úy

  • Thành viên
  • 659 Bài viết

Căn cứ nào mà lại có thể giả sử như thế !!? Bạn chứng minh được điều đó không ??

Bởi vì khi chuyển vế ta được $a+b=\frac{ab}{c}$. Đến đây chắc bạn có thể hiểu lí do vì sao mình đặt như vậy rồi chứ... :closedeyes:  :icon6:


:ukliam2: TOPIC SỐ HỌC - Bachhammer :ukliam2: 

Topic số học, các bài toán về số học

:namtay  :namtay  :namtay  :lol:  :lol:  :lol:  :lol:  :excl:  :excl:  :excl:  :lol:  :lol:  :lol: :icon6:  :namtay  :namtay  :namtay  


#115
Kool LL

Kool LL

    Sĩ quan

  • Thành viên
  • 370 Bài viết

Bài 27: Cho 2 bộ ba số nguyên dương a, b, c và d, e, f sao cho $(a,b,c)=1;(d,e,f)=1$ và thoả mãn đẳng thức $\frac{1}{a}+\frac{1}{b}=\frac{1}{c}$; $\frac{1}{d}+\frac{1}{e}=\frac{1}{f}$. Chứng minh rằng 2(a + b + d + e) là tổng bình phương của hai số tự nhiên.  

 

 

Vì (a, b, c) = 1 tức là $a, b, c$ ko có nhân tử chung thế nên ta hoàn toàn có thể giả sử $b\vdots c,(a,c)=1$...

 

Bởi vì khi chuyển vế ta được $a+b=\frac{ab}{c}$. Đến đây chắc bạn có thể hiểu lí do vì sao mình đặt như vậy rồi chứ... :closedeyes:  :icon6:

 

Nhưng mà với $(a,b,c)=1,\ a+b=\frac{ab}{c},\ ab\ \vdots\ c$ thì sao có thể giả sử $b\ \vdots\ c\ ,\ (a,c)=1$ ???

$(a,b,c)=(10,15,6)$ cũng thoả $(a,b,c)=1,\ a+b=\frac{ab}{c}$ được mà điều giả sử của bạn thì không !

Xem như mình ngu muội, xin bạn hãy giải thích rõ thêm. Vì điều đó thật sự không chặt chẽ chút nào !

 

Giải PT nghiệm nguyên $a,b,c$ thoả : $(a,b,c)=1$ (1) và $\frac{1}{a}+\frac{1}{b}+\frac{1}{c}$ (2)

 

PT này có VSN dạng $(a,b,c)=(\ m(m+n), n(m+n), mn\ )$ với $(m,n)=1$.

Chứng minh:

$(2)\Leftrightarrow a+b=\frac{ab}{c}$. Gọi $m=(a,c)\Leftrightarrow\begin{cases}a=mk\\ c=mn\end{cases}\ (k,n)=1$.

Suy ra $a+b=\frac{kb}{n}\xrightarrow{(k,n)=1} b\ \vdots\ n\Rightarrow b=tn$. Suy ra $mk+tn=kt$ (*)

Do $1=(a,b,c)=(mk,tn,mn)\Rightarrow (t,m)=1$.

$(*)\Rightarrow tn\ \vdots\ k$ và $mk\ \vdots t$ $\xrightarrow{(k,n)=(t,m)=1} t\ \vdots\ k$ và $k\ \vdots\ t$ $\Rightarrow t=k\overset{(*)}{\Rightarrow} m+n=k=t$.

Vậy $(a,b,c)=(\ m(m+n),n(m+n),mn\ )$ với $(m+n,n)=(m+n,m)=1$ tức là $(m,n)=1$.

 

Trở lại Bài 27 :

Theo trên, ta có $a+b=m(m+n)+n(m+n)=(m+n)^2=x^2$.

Tương tự ta cũng có $d+e=y^2$.

Vậy $2(a+b+c+d)=2(x^2+y^2)=(x+y)^2+(x-y)^2$. (đpcm)


Bài viết đã được chỉnh sửa nội dung bởi Kool LL: 06-09-2013 - 20:32


#116
LNH

LNH

    Bất Thế Tà Vương

  • Hiệp sỹ
  • 581 Bài viết

Bài 50: Tìm tất cả các số nguyên dương $m$ và $n$ sao cho

$\left \lfloor \frac{m^2}{n} \right \rfloor+\left \lfloor \frac{n^2}{m} \right \rfloor=\left \lfloor \frac{m}{n}+\frac{n}{m} \right \rfloor+mn$



#117
maitienluat

maitienluat

    Trung sĩ

  • Thành viên
  • 182 Bài viết

Bài 50: Tìm tất cả các số nguyên dương $m$ và $n$ sao cho

$\left \lfloor \frac{m^2}{n} \right \rfloor+\left \lfloor \frac{n^2}{m} \right \rfloor=\left \lfloor \frac{m}{n}+\frac{n}{m} \right \rfloor+mn$

Ta có nhận xét sau: Cho $n$ là một số tự nhiên và $a$ là một số thực. Khi đó nếu $a \geq n$ thì $\left \lfloor a \right \rfloor\geq n$ và nếu $a \leq n$ thì $\left \lfloor a \right \rfloor\leq n$

Trở lại bài toán. KMTQ giả sử $m \geq n$

$\bullet$ TH1: $n^2 \geq m$

Khi đó ta có $(m^2-n)(n^2-m)\geq 0 \Rightarrow mn+1 \geq \frac{m^2}{n}+\frac{n^2}{m}$

Suy ra $\left \lfloor \frac{m^2}{n} \right \rfloor+\left \lfloor \frac {n^2}{m} \right \rfloor=mn+\left \lfloor \frac{m}{n}+\frac{n}{m} \right \rfloor \geq mn+2 \geq \frac {m^2}{n}+\frac{n^2}{m}+1>\left \lfloor \frac {m^2}{n}+\frac {n^2}{m} \right \rfloor \geq \left \lfloor \frac{m^2}{n} \right \rfloor+\left \lfloor \frac {n^2}{m} \right \rfloor$.

Vây trong trường hợp này pt vô nghiệm

$\bullet$ TH2: $n^2 < m$

Đặt $m=an^2+b, b^2=cn+d$. Bỏ qua truờng hợp đơn giản khi $b=0$.

Khi đó ta dễ dàng có được :

$\left \lfloor \frac{m^2}{n} \right \rfloor=a^2n^3+2anb+c; \left \lfloor \frac{n^2}{m} \right \rfloor=0$

Mặt khác để ý rằng $b^2<(c+1)n \Rightarrow b<\sqrt{(c+1)n}\leq \frac {n+c+1}{2} \Rightarrow b \leq \frac {n+c}{2}$

ta dễ dàng chứng minh được $\frac{m}{n}+\frac{n}{m}=n+\frac{(2a-1)bn^2+b^2+n^2}{an^3+bn}< n+c+1 \Rightarrow \left \lfloor \frac{m}{n}+\frac{n}{m} \right \rfloor \leq n+c$

Suy ra $a^2n^3+2abn+c\leq n+c+(an^2+b)n \Leftrightarrow (a^2-a)n^3+(2ab-b-1)n \leq 0$

Mà do $a,b \geq 1$ nên $a^2-a \geq 0$ và $2ab-b-1 \geq 0$ nên ta suy ra $\left\{\begin{matrix} a^2-a=0 & \\ 2ab-b-1=0 & \end{matrix}\right.\Rightarrow \left\{\begin{matrix} a=1 & \\ b=1 & \end{matrix}\right.$. Thử lại thấy $(m,n)=(k^2+1,k)$ thỏa.

KL: $(m,n)=(k^2+1,k);(k,k^2+1)$ với $k \in \mathbb{N}$.


Bài viết đã được chỉnh sửa nội dung bởi maitienluat: 12-09-2013 - 19:18


#118
LNH

LNH

    Bất Thế Tà Vương

  • Hiệp sỹ
  • 581 Bài viết

Bài 51: Cho số nguyên dương $n$ có dạng $n=2^k+1$, $k>1$. CMR: điều kiện cần và đủ để $n$ là số nguyên tố là tồn tại a sao cho $a^{\frac{n-1}{2}}+1 \vdots n$

P/s: mình có ý kiến thế này: bây giờ topic này đã đủ 50 bài rồi (và có lời giải đầy đủ), ta có thể làm một tập san các bài toán trên topic này như một tập tài lieu được không? :))


Bài viết đã được chỉnh sửa nội dung bởi lenhathoang1998: 17-09-2013 - 19:54


#119
Zaraki

Zaraki

    PQT

  • Phó Quản lý Toán Cao cấp
  • 4273 Bài viết


Bài 51: Cho số nguyên dương $n$ có dạng $n=2^k+1$, $k>1$. CMR: điều kiện cần và đủ để $n$ là số nguyên tố là $a^{\frac{n-1}{2}}+1 \vdots n$

P/s: mình có ý kiến thế này: bây giờ topic này đã đủ 50 bài rồi (và có lời giải đầy đủ), ta có thể làm một tập san các bài toán trên topic này như một tập tài lieu được không? :))

^_^

Cho em hỏi điều kiện của $a$ là gì nhỉ ??


  • LNH yêu thích

Discovery is a child’s privilege. I mean the small child, the child who is not afraid to be wrong, to look silly, to not be serious, and to act differently from everyone else. He is also not afraid that the things he is interested in are in bad taste or turn out to be different from his expectations, from what they should be, or rather he is not afraid of what they actually are. He ignores the silent and flawless consensus that is part of the air we breathe – the consensus of all the people who are, or are reputed to be, reasonable.

 

Grothendieck, Récoltes et Semailles (“Crops and Seeds”). 


#120
LNH

LNH

    Bất Thế Tà Vương

  • Hiệp sỹ
  • 581 Bài viết

^_^

Cho em hỏi điều kiện của $a$ là gì nhỉ ??

Ở đây chỉ nói là tồn tại $a$ thôi







Được gắn nhãn với một hoặc nhiều trong số những từ khóa sau: topic số học, hay, tuyệt

0 người đang xem chủ đề

0 thành viên, 0 khách, 0 thành viên ẩn danh